NCSBN Practice Questions 61-75

Pataasin ang iyong marka sa homework at exams ngayon gamit ang Quizwiz!

A client is to begin taking alendronate. Which of these instructions should the nurse emphasize when teaching about this medication? A. "Take the medication with a full glass of milk two hours after meals." B. "It is recommended that you take this medication with calcium and a glass of juice." C. "Be sure to take this medication on an empty stomach." D. "You may take this medication after any meal, at the same time every day."

C Alendronate (Fosamax) is used to treat and prevent osteoporosis. It should be taken first thing in the morning with 6 to 8 ounces of plain water at least 30 minutes before other medication or food. Food and fluids (other than water) greatly decrease the absorption of this medication. The client must also be instructed to remain in the upright position for 30 minutes following the dose to facilitate passage into the stomach and minimize irritation of the esophagus.

A nurse is caring for a client who was in a motor vehicle accident. Which of these findings would be the highest priority if newly identified by the nurse? A. Flaccid paralysis B. Reduced sensory responses C. Pupils fixed and dilated D. Diminished spinal reflexes

C Pupils that are fixed and dilated indicate overwhelming injury and intrinsic damage to the upper brain stem, and would be the highest priority as it is a poor prognostic sign. The other findings are more consistent with partial dysfunction of the brain or spinal cord.

A nurse often works with many clients from different cultures. Which approach is a priority for the nurse? A. Speak at least two other languages of clients in the neighborhood B. Recognize personal attitudes about cultural differences along with any real or expected biases C. Learn about the cultures of clients who are most often encountered D. Have a list of persons for referral when interaction with clients from different cultures occur

B A nurse must discover personal attitudes, prejudices and biases specific to different cultures. Awareness of these will prevent negative consequences for interactions with clients and families across various cultures.

A client diagnosed with an aplastic sickle cell crisis is within the initial 10 minutes of receiving a blood transfusion. The client reports "feeling hot." Almost immediately, the client begins to have audible wheezes. What should be the nurse's first action? A. Stop and disconnect the blood infusion B. Take and record vital signs C. Notify the health care provider D. Send blood samples to the lab

A If a reaction of any type is suspected during administration of blood products, stop the infusion immediately, disconnect the blood product line and connect a line with 0.9% normal saline at a keep open rate, notify the health care provider, monitor the vital signs and any other changes, and then send a urine and blood sample to the lab.

The nurse recognizes that which finding indicates a child has attained the developmental stage of concrete operations, according to Piaget? A. The child makes the moral judgment that "stealing is wrong." B. The child thinks in mental images or word pictures. C. The child explores the environment with the use of sight and movement. D. The child reasons that homework is time-consuming but necessary.

A The stage of concrete operations is characterized by logical thinking and moral judgments. This stage is associated with school-aged children from about age 7 to 11. Exploring the environment is seen in the sensorimotor stage (birth to 24 months). Mental symbolization is seen in the preoperational stage (2 to 4 years). Formal operational thought is seen with adolescents, who might reason that homework is necessary.

The nurse is working to establish a therapeutic relationship with a client. Which of these approaches would be most damaging to the client's self-esteem? A. Indifference B. Fear C. Disapproval D. Anger

A Therapeutic relationships that build or maintain self-esteem are incongruent with indifference. Positive connectedness or caring characterizes a therapeutic relationship, which will enhance self-esteem.

A parent asks the school nurse how to eliminate lice from a child's head. What is the appropriate response by the nurse? A. Apply a pediculicide as directed B. Apply warm soaks to the head twice daily C. Wash the child's linen and clothing in a bleach solution D. Cut the child's hair short to remove the nits

A Treatment of head lice usually consists of an application of a pediculicide. Parents should be sure to follow the product directions. It is important that parents understand that no product is 100% ovicidal and, consequently, some nits will survive. Parents will need to use a nit comb to remove any surviving nits that cling to the hair shaft. In order to avoid reinfestation, bed linens must be washed in hot water and dried in the dryer; toys and objects that cannot be washed should be bagged.

During administration of medications to a client, the client says to a nurse, "I do not want to take that medicine today." Which of these statements should the nurse use as a response? A. "Is there any particular reason why you don't want to take your medicine?" B. "I will have to call your doctor and report this." C. "Do you understand the consequences of refusing your prescribed treatment?" D. "That's OK, its all right to skip your medication now and then."

A When a new problem is identified, it is important for the nurse to collect accurate information directly from clients. This is crucial to ensure that clients' needs are adequately identified in order to select the best nursing care approaches. The nurse should pursue a conversation with the client to reveal any reasons for the medication refusal. It may be that the client has developed untoward side effects.

The nurse is caring for a client admitted to the hospital with severe left-sided flank pain and hematuria. Diagnostic tests indicate a kidney stone partially obstructing the left ureter. Which of the following outcomes is the most important for this client? A. Adequate urinary elimination is maintained B. Verbalizes understanding of the disease process C. Pain controlled with medication D. Tolerates diet without nausea and vomiting

A While all options are appropriate to the care of this client, urinary elimination is the nursing priority. A stone that completely obstructs the ureter can cause hydronephrosis and potential kidney damage. Remember Maslow - physiologic needs are more important than nutritional needs. Pain control and teaching are lower priorities.

The nurse is caring for a 10 year-old child who is diagnosed with diabetes insipidus (DI) and is receiving vasopressin. What is the priority for the nurse to teach the child and the family members about this prescribed medication? A. The child will need intravenous therapy for several weeks B. The family must monitor the child for arrhythmias C. Parents should administer the daily intramuscular injections D. The family must observe the child for dehydration

B Diabetes insipidus is characterized by a decreased secretion of antidiuretic hormone (ADH). Decreased ADH results in polyuria and polydipsia; the person is unable to concentrate urine. Vasopressin is the drug of choice to treat central DI. At home, it can be administered 2-3 times a day, either IM, subQ or intranasally. Not drinking enough fluids can cause arrhythmias, fatigue and muscle pain. Other serious side effects include chest pain, skin discoloration and paresthesia.

The nurse is teaching the parents of a client diagnosed with sickle cell anemia. What should the nurse discuss with the parents about their child's condition? A. There's a depression of the platelets and also the red and white blood cells B. There is a reduced number of red blood cells due to inadequate iron in the diet C. Red blood cells are abnormally shaped, preventing adequate oxygen delivery to the tissue D. Sickle-shaped red blood cells carry carbon dioxide to the tissues instead of oxygen.

C Sickle cell anemia is caused by an abnormal type of hemoglobin, which changes the shape of red blood cells from a round to a sickle shape. These fragile abnormal blood cells carry less hemoglobin and can get stuck in the smaller blood vessels, depriving the tissue of oxygen and causing severe pain and tissue damage. Inadequate dietary iron causes iron-deficiency anemia. Platelets, red blood cells and white blood cells are all depressed in pancytopenia.

A nurse is taking a health history from a Native American client. It is critical that the nurse remember that eye contact with such clients is considered as which behavior? A. Expected B. Professional C. Enjoyable D. Rude

D Native Americans consider direct eye contact to be impolite or aggressive among strangers.

The RN is caring for a client immediately after a cholecystectomy. Which of these tasks can the RN safely ask an unlicensed assistive personnel (UAP)? A. Document amount of output into the drainage collection device B. Document amount of drainage on the surgical dressing C. Document changes in abdominal distention D. Assess the return of bowel sounds or passing flatus

A The emptying, measuring and recording drainage from a postoperative drain may be delegated to the UAP who has demonstrated competence in performing this task. While the RN is responsible for all care-related decisions, the UAP can typically perform tasks that have predictable outcomes. The other tasks or activities required nursing knowledge, skill or judgment and cannot be assigned to the UAP.

A nurse is performing a physical assessment on a toddler. Which approach should be the first one to use with this age client? A. Use minimal physical contact initially in the exam B. Proceed from head to toe in a sequential manner C. Explain the exam in detail as areas are examined D. Perform traumatic procedures first

A The nurse should approach a toddler slowly and use minimal physical contact initially so as to gain the toddler's cooperation. Other approaches with this age group are to be flexible in the sequence of the exam and give only brief simple explanations just prior to any action.

The nurse is caring for a client with Legionnaire's disease. Which finding would require the nurse's immediate attention? A. Decreased chest wall expansion B. A decrease in respiratory rate from 34 to 24 C. Dry mucus membranes in the mouth D. Pleuritic pain on inspiration

A The respiratory status of a client with this acute bacterial pneumonia known as Legionnaires' disease is critical. Note that all of these findings would be of concern, but a decrease in chest wall expansion is the priority because it reflects a possible decrease in the depth and effort of respirations. Further findings of restlessness, including low oxygen saturation, would indicate hypoxemia. The client may need to have oxygen titrated to maintain adequate O2 saturation. Mechanical ventilation may be needed for signs of respiratory failure.

A mother brings her 26 month-old to the well-child clinic and expresses frustration and anger due to the child constantly saying "no" and refusing to follow directions. The nurse should explain that this is normal for the age, as negativism is an attempt to meet which developmental need? A. Initiative B. Independence C. Trust D. Self-esteem

B In Erikson's theory of development, toddlers struggle to assert independence. They often use the word "no" even when they mean yes. This stage is called autonomy versus shame and doubt. At this stage other characteristics are "grazing" instead of eating a meal, a behavior of rituals especially at bedtime and parallel play with other children.

When admitting a client to the ambulatory surgery unit, the nurse notices the client has painted fingernails. The nurse reviews the pre-op orders and notes that pulse oximetry is prescribed. Which statement by the nurse is appropriate? A. "I will ask your provider if we must ruin those beautiful nails." B. "So that we can measure your oxygen levels, may I please remove the polish from at least two nails?" C. "I am sorry. All your nail polish must be removed." D. "If you do not remove all your polish, I will request a needlestick to test oxygen levels."

B In order to effectively measure pulse oximetry, there can be no nail polish on the finger fitted with the reading device. The client should be approached using therapeutic communication skills. The other options are inappropriate.

The hospital staff requests that parents who have a Greek heritage remove the amulet from around their infant's neck. The parents refuse. The nurse should understand that the parents may be concerned about which factor? A. Fright from spiritual beings B. Evil eye or envy of others C. Balance in body systems D. Mental development delays

B In the Greek heritage the matiasma, "bad eye" or "evil eye, " results from the envy or admiration of others. The belief is that the eye is able to harm a wide variety of things, including inanimate objects and that children are particularly susceptible to attacks. Persons of Greek heritage employ a variety of preventive mechanisms to thwart the effects of envy. One of these is the protective charm in the form of an amulet that consists of blessed wood or incense.

The nurse is providing diet instruction to the parents of a child with cystic fibrosis. The nurse would emphasize that the diet should follow which of the following guidelines? A. Sodium-restricted B. High-fat, high-calorie foods C. Skim milk and low-fat dairy products D. Restricted calorie

B The child with cystic fibrosis requires a well-balanced diet that is high in calories (approximately 2,900 to 4,500 calories a day). The diet should include increased amounts of protein, iron, salt, zinc and calcium (especially full-fat dairy products.) Fat does not need to be restricted because these children lose fat in the stool. Recall one of the characteristics of this disease is fatty, foul smelling stool.

A client is being maintained on heparin therapy for deep vein thrombosis (DVT). A nurse must closely monitor which of these following laboratory values? A. D-dimer B. Platelet count C. Activated partial thromboplastin time D. Bleeding time

C Heparin is used to prevent further clots from being formed and to prevent the present clot from enlarging. The activated partial thromboplastin time (APTT) test measures the time it takes blood to clot and is used to monitor the effectiveness of heparin therapy. The therapeutic range is about 1 1/2 to 2 or 2 1/2 times the normal values. D-dimer is used to evaluate blood clot formation. Platelet counts are used to evaluate abnormal bleeding times. Bleeding time refers to the time it takes for a pinprick to stop bleeding (normally, about 2 1/2 minutes.)

The client has been taking isoniazid (INH) and rifampin for several months to treat pulmonary tuberculosis. Which laboratory tests does the nurse anticipate will be ordered for this client. A. Cardiac enzymes B. Pancreatic enzymes C. Liver enzymes D. Kidney function

C INH, as well as other long-term by-mouth medications, can cause hepatocellular injury. The nurse anticipates monitoring liver enzymes (ALT, AST and alkaline phosphatase) because these are released into the blood when the liver is damaged.

A child is admitted to the pediatric unit with a diagnosis of suspected meningococcal meningitis. Which admission orders should the nurse implement first? A. Institute seizure precautions B. Administer cefotaxime (Claforan) IV 50 mg/kg/day divided every six hours C. Initiate droplet precautions D. Monitor neurologic status every hour

C Meningococcal meningitis is a bacterial infection that can be communicated to others. The initial therapeutic management of acute bacterial meningitis includes droplet precautions, initiation of antimicrobial therapy, monitoring neurological status along with vital signs, instituting seizure precautions and, lastly, maintaining optimum hydration. The first action is to initiate any necessary precautions to protect themselves and others from the potential infection. Viral meningitis usually does not require protective measures of isolation and these clients often return home to recover.

A 72 year-old client is admitted for possible dehydration. The nurse knows that older adults are particularly at risk for dehydration due to which physiologic change? A. Higher metabolic demands B. An increased need for extravascular fluid C. A decreased sensation of thirst D. An increase in diaphoresis

C Older adults have a reduction in thirst sensation and this causes them to consume less fluids. Other risk factors may include fear of incontinence, inability to drink fluids independently, increased frequency to void with increased fluid intake, and lack of motivation.

The parents of a 4 year-old child recovering from chickenpox (varicella) would like the child to return to day care as soon as possible. In order to ensure that the illness is no longer communicable, what should the nurse assess for in this child? A. Rhinorrhea and coryza B. Presence of vesicles C. All lesions crusted D. Elevated temperature

C The chickenpox rash begins as a macule, with fever, and progresses to a vesicle that breaks open and then crusts over. When all lesions are crusted, the child is no longer in a communicable stage.

The nurse is using the SBAR technique to communicate with the health care provider. Which of the following phrases would be associated with "B-Background"? A. "Vital signs are..." B. "I would like you to..." C. "The client's treatments are..." D. I'm not sure what the problem is, but the client's condition is deteriorating."

C The correct option gives the health care provider background information about the client, including age, primary diagnosis, treatments, etc. Stating that the client's condition is deteriorating is the situation (S). Stating, "I would like you to..." is the request or recommendation (R). Vital signs are part of the assessment (A). Using SBAR is an effective technique used to improve communication with other members of the health care team. This, in turn, helps to foster a culture of safety.

A 72-year old client reports having discomfort immediately after a below-the-knee amputation. Which initial action by the nurse is most appropriate? A. Administer opioid narcotics as ordered B. Conduct guided imagery or distraction C. Wrap the stump snugly in an elastic bandage D. Ensure that the stump is elevated

D Elevating the stump is the priority intervention for the first 24 hours after surgery. This will help prevent pressure due to postoperative swelling, which will minimize pain or discomfort. Without this action, a firm elastic bandage, opioid narcotics, or guided imagery will have little effect. Analgesics appropriate to the level of pain should be administered as needed in the postoperative period to promote client comfort. After the first day, the residual limb should be flat on the bed.

The nursing student assigns an unlicensed assistive person (UAP) to complete several tasks. After reviewing the assignments with a preceptor, it is determined that one of the tasks cannot be performed by an UAP. Which task must be reassigned to a registered nurse (RN)? A. Ambulate the client in the hallway twice a shift B. Provide information about a low-sodium diet prior to discharge C. Assist a client in skeletal traction with meals and snacks D. Obtain a daily weight on a client before breakfast

B The focus of this question is to select the incorrectly assigned task. UAP are typically assigned tasks with predictable outcomes. They assist with activities of daily living, collect specimens, measure weight and can assist with ambulation. It is the registered nurse (RN), and not the UAP, who can teach, initiate referrals or conduct evaluations, as in the case of the client who is being prepared for discharge.

A woman who is 26-weeks pregnant is admitted with painless vaginal bleeding. The nurse should prepare the client for which of these procedures or tests? A. Non-stress test B. Transvaginal ultrasound C. Leopold maneuvers D. Serum hCG level

B The most common cause of painless vaginal bleeding involves a problem with the placenta, such as placenta previa. A transvaginal ultrasound (an ultrasound device, inserted into the vagina) is the test of choice to confirm placenta previa. Serum hCG is used to screen for a pregnancy. A fetal non-stress test is used in pregnancies over 28 weeks to measure fetal heart rate and contractions. Leopold maneuvers is a stepwise method of abdominal palpation used after about 34 weeks gestation to determine fetal lie and presentation.

The nurse is providing instructions for a client with asthma. Which of these factors is a priority for the client to monitor daily? A. Respiratory rate B. Peak air flow volumes C. Respiratory effort D. Pulse oximetry

B The peak airflow volume decreases about 24 hours before clinical manifestations of exacerbation of asthma. Note that the question asks for a priority so all of the options would be monitored. However, the peak air flow is the priority.

The mother of a 2 year-old hospitalized child asks the nurse's advice about the child's screaming every time the mother gets ready to leave the hospital room. What is the best response by the nurse? A. "You might want to "sneak out" of the room once the child falls asleep." B. "Keep in mind that for the age this is a normal response to being in the hospital." C. "Oh, that behavior will stop in a few days with patience from you." D. "I think you or your partner needs to stay with the child while in the hospital."

B The protest phase of separation anxiety is a normal response for a child this age. In toddlers, ages one to three, separation anxiety is at its peak. After three years of age it begins to diminish until the adolescent years, when the behavior is minimal.

A client is diagnosed with a spontaneous pneumothorax that requires the insertion of a chest tube with a flutter valve. What is the best explanation that the nurse should provide to the client? A. "The tube will drain fluid from your chest." B. "The excess air from your chest will be removed by the tube." C. "The amount of air that enters your chest will be controlled by the flutter valve." D. "The hole in your lung will be sealed with this tube insertion."

B The purpose of the chest tube is to create negative pressure to allow the passive removal of the air that has accumulated in the pleural space. The flutter valve is a one way valve that allows the air to leave the pleural space. It blocks any air reentry. The use of a flutter valve with a spontaneous pneumothorax allows for increased mobility without the use of a chest seal drainage system.

A client states, "People think I'm no good, you know what I mean?" Which of these responses by the nurse would be the most therapeutic? A. "I think you're good. So you see, there's one person who likes you." B. "I'm not sure what you mean. Tell me a bit more about that." C. "Let's discuss this to see the reasons you create this impression on people." D. "Well people often take their own feelings of inadequacy out on others."

B This therapeutic communication technique elicits more information, especially when delivered in an open, nonjudgmental fashion. The use of the nursing process when a client has a problem is "further assessment" of the situation.

During medication reconciliation, the client's partner explains that the client stopped taking the prescribed sertraline and started using the partner's tranylcypromine to treat his depression. The client is now experiencing "twitching muscles" and a "racing heart." What additional adverse reaction should the nurse immediately assess for and alert the health care provider about? A. Pulmonary edema B. Mental status changes C. Muscle weakness D. Atrial fibrillation

B Use of serotonergic agents may result in serotonin syndrome with confusion, nausea, palpitations, increased muscle tone with twitching muscles and agitation. Serotonin syndrome is most often reported in clients taking two or more medications that increase CNS serotonin levels by different mechanisms. The most common drug combinations associated with serotonin syndrome involve the MAOIs, SSRIs, and the tricyclic antidepressants.

A nurse is caring for a 69-year-old diagnosed with hyperglycemia. Which activity or task could be assigned to the unlicensed assistive person (UAP)? A. Review the initial signs of hyperglycemia with the client's family B. Check the condition of the skin of the lower extremities C. Monitor for altered levels of consciousness (LOC) D. Record dietary intake

D The UAP can perform routine activities with predictable outcomes, such as recording dietary intake. Although the UAP can usually assist clients with personal hygiene and would be able to identify a change in LOC (for example, the client does not respond appropriately to questions), their role is to inform the nurse about changes in the client's condition. The nurse must follow up on this information and perform a focused assessment, communicate changes in the client's condition with the health care team and then develop a revised plan of action for client care.

The nurse is auscultating the heart of a client who is diagnosed with dilated cardiomyopathy. What finding would the nurse expect to hear? A. Diastolic murmur B. Apical click C. Ventricular gallop of S3 D. Split S2

C A ventricular gallop, S3 is caused by blood flowing rapidly into a distended noncompliant ventricle. This is the most common sound with left-sided heart failure. It sounds like "Kentucky." Increased left heart pressures may cause dilation of the mitral valve in the client with heart failure resulting in a systolic murmur.

A nurse is teaching the parents of a 3 month-old infant about nutrition. What is the main source of fluids for an infant until about 12 months of age? A. Room temperature fruit juice B. Diluted nonfat dry milk C. Formula or breast milk D. Fluoridated tap water

C Formula or breast milk are the perfect food and source of nutrients and liquids up to 1 year-old. Juice should not be used as the main fluid.

A nurse is caring for an acutely ill 10 year-old child. Which assessment finding would require the nurse's immediate attention? A. Profuse diaphoresis B. Temperature of 101.3 F (38.5° C) C. Rapid bounding pulse D. Slow, irregular respirations

D A slow and irregular respiratory rate is a sign of respiratory fatigue and failure in an acutely ill child. Respiratory failure can rapidly lead to respiratory arrest. Emergency intervention for respiratory support is indicated.

A nurse is teaching a school-age child and family members about the use of inhalers prescribed for asthma. What is the best way to evaluate the effectiveness of the treatments? A. Observe use of peak flow meter B. Rely on child's self-report C. Monitor pulse rate D. Note skin color changes

A The peak flow meter can help determine if the symptoms of asthma are in control or are worsening. It works by measuring how fast air comes out of the lungs when the client forcefully exhales (the peak expiratory flow or PEF) after inhaling fully. The client should record the highest of three readings in an asthma diary. Children ages 4 and up should be able to use a peak flow meter.

The nurse is caring for a victim of domestic abuse. Which characteristic is commonly associated with the abuser? A. Alcohol addiction B. Overconfidence C. High tolerance for frustration D. Low self-esteem

D Abusers are often charming, jealous, manipulative, controlling, narcissistic, inconsistent, critical, hypersensitive, vicious and cruel. Even though a lot of abusers seem "tough" and "confident," they often suffer from low self-esteem. Alcohol can make someone who is easily frustrated and angry more violent, but alcoholism does not cause abuse.

A child with tetralogy of Fallot visits the clinic several weeks before the scheduled surgery. The nurse should give priority attention to which focus? A. Observation for developmental delays B. Prevention of infection C. Maintenance of adequate nutrition D. Assessment of oxygenation

D All of the responses would be important for a child diagnosed with tetralogy of Fallot. However, persistent hypoxemia causes acidosis, which further decreases pulmonary blood flow. Additionally, low oxygenation leads to development of polycythemia and may result in neurological complications.

There is an order to administer atropine to a client preoperatively. The nurse understands that which effect is the intended purpose for giving this medication preoperatively? A. Reduce heart rate B. Elevate blood pressure C. Enhance sedation D. Decrease secretions

D Atropine is a common anesthesia adjunct. It decreases the amount of secretions which, in turn, decreases the risk of aspiration during the operative procedure.

While performing an initial assessment on a newborn following a breech delivery, the nurse suspects hip dislocation. Which of these findings is most suggestive of this abnormality? A. Flexion of lower extremities B. Lengthened leg of affected side C. Negative Ortolani response D. Irregular hip symmetry

D Early assessment of irregular hip symmetry alerts the nurse and the provider to a correctable congenital hip dislocation. The leg is shortened on the affected side. One check for hip dislocation is the Ortolani click; if it is found, it is called a positive response.

The nurse is planning care for an 8 year-old child. Which approach should be included in the plan of care? A. Provide frequent reassurance and cuddling as nonverbals indicate B. Encourage the child to engage in activities while in the playroom C. Promote dependence for activities of daily living D. Talk with the child to stimulate the child to express opinions

B According to Erikson, school-age children are in the stage of industry versus inferiority. To help them achieve industry, the nurse should encourage this group of children to carry out tasks and activities in their rooms or while in the playroom.

Following craniotomy surgery the client develops a cardiac arrhythmia, and the provider orders lidocaine (Xylocaine) infusion at 3 mg/minute. The label states the 500 mL IV bag contains 2 grams of lidocaine. What is the flow rate setting (milliliter/hour)? (Round to the nearest whole number and write only the number.)

45 Using dimensional analysis to solve, because the final answer will be in mL/hour, begin the equation with milliliters on top. Multiply by known factors to cancel out unwanted units until only mL/hour remains.(500 mL/2 gram) X (1 gram/1000 mg) X (3 mg/min) X (60 min/hr) = 90,000/2,000 = 45 mL/hour.

An oncology client has developed an infection, and there is now an order for a 50 mL bag of IV antibiotics to infuse over 15 minutes. After 5 minutes, 40 mL remains in the bag. The package insert indicates the IV tubing delivers 15 gtt/mL. Calculate the flow rate in drops/minute (gtt/min) to deliver the volume on time. (Round to the nearest whole number and write only the number.)

60 To solve, recognize what is really asked. There are 40 mL remaining and 10 more minutes to get it infused. If you use dimensional analysis to solve the problem, and the final answer is in gtt/min, begin the equation with gtts on top, then multiply to cancel out unwanted units (mL) until the answer (gtt/min) remains. 15 gtt/1 mL X 40 mL/10 min = 600/10 = 60

A 187-pound client with a subdural hematoma and findings of increased intracranial pressure has been prescribed 25% solution mannitol 0.25 g/kg to be administered by intravenous push right away. The pharmacy has sent up four 50 mL bottles (12.5 g/50 mL is written on the label). How many milliliters should the nurse prepare to give the client? (Write the answer using whole numbers.)

85 Convert pounds to kilograms, calculate the dose this client requires based on his weight.Convert from pounds to kilograms: 187 lbs/2.2 = 85 kg0.25 g x 85 kg = 21.25 g(12.5 g/50 mL) = (21.25 g/x mL)x = 1062.50/12.5 = 85 mLAn alternate method for solving the problem is to use dimensional analysis. Because the answer will be milliliters, begin the equation with milliliters on top, then multiply to cancel unwanted units until only the milliliters remain.(50 mL/12.5 g) X (0.25 g/kg) X (1 kg/2.2 lbs) X (187 lb/1) = 85

The unlicensed assistive personnel (UAP) reports to the nurse that the oral temperature of a post-surgical client has increased from 99 F (37.2 C) at 5:00 pm to now 101 F (38.3 C) at 6:00 pm. The nurse walks into the room to check the client and observes a cup of steaming, hot coffee at the bedside. What instructions are appropriate to give to the UAP? A. Recheck the oral temperature 15 minutes after removing the hot coffee from the bedside B. Encourage the client to drink more oral fluids to prevent dehydration C. Chart this temperature elevation on the flow sheet and retake the temperature in 2 hours D. Provide the client with only cold water and juices to drink every hour

A A recheck of the oral temperature is needed to eliminate the possibility of an artificial elevation of temperature from the hot coffee. Hot or cold liquids, smoking, eating, chewing gum, and talking can all elevate or lower the temperature if done within 10 minutes of the temperature being taken. Waiting to take the temperature for 15 minutes will help the temperature return to its normal reading and facilitate an accurate reading. The nurse should avoid premature assumptions about explanations for findings and the initial action is to do an assessment of the client.

An important goal in the development of a therapeutic inpatient milieu is which of these items? A. Provide a testing ground for new patterns of behavior while clients take responsibility for their own actions B. Discourage expressions of anger because such feelings can be disruptive to other clients C. Form a group forum in which clients decide on unit rules, regulations and policies D. Offer a businesslike atmosphere where clients can work on individual goals

A A therapeutic milieu is purposeful and planned to provide safety and a testing ground for new patterns of behavior. The other approaches may be part of other types of therapies.

The client is withdrawn. Which nursing intervention will be most effective to help the client develop relationship and interpersonal skills? A. Offer the client frequent opportunities to interact with one person B. Assist the client to analyze the meaning of the withdrawn behavior C. Discuss with the client the focus that other clients have similar problems D. Provide the client with frequent opportunities to interact with other clients

A A withdrawn client is uncomfortable in social interaction. The nurse-client or a one-on-one relationship is a corrective relationship in which the client learns tolerance and skills for relationships.

The nurse is suctioning a client's tracheostomy. During this procedure, the nurse should instill saline for what purpose? A. Facilitate the removal of mucus plugs B. Decrease the client's discomfort C. Reduce the viscosity of secretions D. Prevent client aspiration from secretions

A According to evidence-based practice, the use of saline is no longer recommended during routine suctioning. However, if a client is suspected to have a mucous plug in the larger bronchial or in an artificial airway (such as a tracheostomy tube), the nurse can instill sterile normal saline to thin and loosen the plug or viscous secretions.

A 3 year-old child has tympanostomy tubes in place. The child's parent asks the nurse if the child can swim in the family pool. How best should the nurse respond? A. "Your child may swim in your own pool but should not dive under the water." B. "Your child may swim anywhere without restrictions." C. "Your child may swim and dive if earplugs are worn when in and around the pool." D. "Your child should not swim in the pool while the tubes are in place."

A After this procedure, the child can swim in the family pool, without earplugs, as long as s/he does not dive under the water. Since lakes and oceans are not as clean as pool water, the child should not put his/her head under the surface of the water unless waterproof ear protection is used. Children should also not submerge their heads under the water in a bathtub. Unless the child develops frequent drainage after swimming or bath, routine use of earplugs are usually not recommended.

The nurse is caring for a client with a diagnosis of gastroesophageal reflux disease (GERD). The primary health care provider's orders include omeprazole (Prilosec) twice a day, Maalox prior to meals, elevation of the head of the bed, acid-reflux diet, and no alcohol. Which one of these orders would the nurse question? A. Schedule for antacid B. Schedule for the proton-pump inhibitor C. Bed position D. Prescribed diet

A All of the options listed are potential recommendations but the schedule for antacids should be one to three hours after eating and at bedtime as needed.

The nurse is caring for a client who has a history of asthma and is now diagnosed with gastroesophageal reflux disease (GERD). Which of these types of medications, which are all prescribed for the client, may aggravate the GERD? A. Anticholinergic bronchodilator B. Anti-infective C. Histamine blocker D. Corticosteroid

A An anticholinergic medication will decrease gastric emptying and diminish the pressure on the lower esophageal sphincter. This will enhance gastric reflux.

A nurse is caring for a client with acute renal failure who has a subclavian vascular access port for hemodialysis. Which of these findings necessitates immediate action by the nurse? A. Elevated temperature B. Dry, hacking cough C. Chronic fatigue D. Pruritic rash

A An elevated temperature in this client would indicate a possible central line infection. This finding should be reported to the provider who should order wound and blood cultures. If a line infection is suspected, the line will need to be removed, necessitating alternate line placement for hemodialysis. Interventions to prevention line infection through maintenance of line sterility and stabilization of the site are a priority in any client with a central line. The other findings should be reported to the health care provider but a febrile reaction is the priority.

A nurse is caring for a client who had a mastectomy two months ago. Which of these statements, made by a client, is incorrect and indicates a need for an additional assessment associated with the impact of an alteration in body image? A. "I only look at myself in the mirror after I am fully dressed." B. "I guess it's time for me to quit wearing a bikini at my age anyway." C. "I plan to volunteer to work with others who have had mastectomies in Reach for Recovery." D. "It really isn't much of a problem for me, I never had large breasts anyway."

A An inability to look at the incision or surgical site is associated with possible denial or anger during the process of coping with a loss. This indicates that a problem area for this client is body image. The other statements reflect movement towards acceptance of the loss of a "normal" figure.

The client, who is 12-hours post gastric bypass surgery, is restless and reports increasing back and shoulder pain unrelieved by pain medication. What action should the nurse take first? A. Report the complaint to the surgeon immediately B. Place the client in Trendelenburg position C. Roll the client to side-lying position to ensure the epidural analgesia catheter is still in place D. Check the nasogastric (NG) tube for patency and reposition the tube

A Anastomotic leak is the most serious complication after bariatric surgery and the most common cause of death. Clients should be monitored for increased back, shoulder, or abdominal pain, restlessness, unexplained tachycardia, and oliguria; these findings should be immediately reported to the surgeon. A nasogastric tube should not be manipulated postoperatively because it could damage the surgical site. In order to reduce intra-abdominal pressure on the diaphragm and improve tidal volumes, the head of the bed should be elevated 30 to 45 degrees, preferably in reverse Trendelenburg position. Patient controlled analgesia is used for pain management; epidural analgesia is not often used because of the difficulty of locating exact spinal segments for proper insertion of a catheter.

The nurse is caring for a client newly diagnosed with atrial fibrillation. The atrial heart rate is 250 and the ventricular rate is controlled at 75. Which finding is a cause for the most concern? A. Difficulty speaking B. Diminished bowel sounds C. Tachypnea with movement D. Loss of appetite

A Anticoagulant therapy is usually given to patients with atrial fibrillation to prevent blood clots and stroke. A new finding of difficulty speaking may indicate that the client has suffered a stroke. The nurse should assess for any other cognitive changes, assess lung function, and immediately contact the provider and possibly the stroke team. The atrial rate of 250 is normal for atrial fibrillation and is of no concern; the ventricular rate of 75 indicates that the cardiac rate is well-controlled (a ventricular rate above 100 would not be adequately controlled, necessitating additional rate control medications such as a beta blocker, calcium channel blocker or digoxin).

A client with a history of heart disease takes prophylactic aspirin daily. What should the nurse monitor to help prevent aspirin toxicity? A. Serum albumin B. Protein intake C. Serum potassium D. Conductive hearing loss

A Aspirin and salicylic acid are bound to serum albumin. A low serum albumin level may result in altered salicylate binding, thereby increasing the availability of unbound (active) drug for toxic effects. The effect is more evident in the elderly, especially someone with heart disease taking other medications that may be albumin-bound. Although aspirin can cause tinnitus and hearing loss, conductive hearing loss is typically the result of ear infections, allergies or tumors, not aspirin toxicity.

A 65-year-old Hispanic-Latino client, who is a practicing Catholic, has been diagnosed with prostate cancer. He adamantly refuses pain medication, believing that suffering is part of life. The client states, "Everyone's life is in God's hands." What should be the next action by a nurse? A. Ask the client if talking with a Catholic priest would be desired B. Document the situation on the notes C. Report the situation to the health care provider D. Discuss the situation with the client's family

A Belief regarding pain are the oldest culturally-related research areas in health care. Astute observations and careful assessments must be completed to determine the level of pain a person can tolerate. Health care practitioners must investigate the meaning of pain to each person within a cultural explanatory framework. The other actions may be done later.

The family of a client diagnosed with schizophrenia is upset to learn that there is an order to transfer the client from a locked inpatient facility to a community group home setting. The family tells the nurse that they need the client stay to stay in the locked facility. Which response by the nurse is most appropriate? A. It violates the principle of least restrictive environment if the client, who qualifies for community placement, is kept in a locked facility B. In keeping with the principle of beneficence, the health care provider decides on which placement is best for the client C. Due to confidentiality issues, you will need to take your concerns directly to the health care provider D. If the client agrees to stay, I will submit a request for voluntary commitment to protect the client's right of self-determination

A Clients have the right to be treated in the least restrictive environment. Additionally, a client cannot be restrained or locked in a facility when less restrictive options are available. A health care provider and nursing team assess a client's ability to function in a less restrictive setting and initiate transfer when appropriate. Ethical principles of beneficence, confidentiality and self-determination are important, but are not the overriding principles in the decision to move a client from a locked facility to a less restrictive, community facility.

During assessment of a postpartum client, the nurse palpates a firm fundus and observes a constant trickle of bright red blood from the vagina. What is the most likely cause of these findings? A. Genital lacerations B. Retained placenta C. Uterine atony D. Clotting disorder

A Continuous trickling of blood in the absence of a boggy fundus indicates undetected genital tract lacerations. The client may need to return to surgery to close up the lacerations.

The nurse is teaching a client diagnosed with type 2 diabetes mellitus about the prescribed diet. What information would the nurse want to include? A. Keep a regular schedule of meals and snacks B. Keep something sweet available at all times for hypoglycemic episodes C. Reduce carbohydrate intake to 25% of total calories D. Maintain previous calorie intake but add more protein

A Currently, calorie-controlled diets with strict meal plans are rarely suggested for clients diagnosed with diabetes mellitus. The proper approach to eating is an incorporation of a schedule with food changes into clients' existing dietary patterns. Clients should learn to read labels and identify specific canned foods, frozen entrees, or other foods that are acceptable or to be avoided.

A client had a full leg plaster cast applied during surgery. What is the priority reason for the nurse to elevate the casted leg postoperatively? A. Improve venous return B. Reduce the drying time C. Promote the client's comfort D. Decrease irritation to the skin

A Elevating the leg on one or two pillows will improve venous return, which will reduce the amount of swelling in the extremity. Ice may be ordered to also help prevent or reduce swelling. Elevating the leg will have little effect on drying time. Skin irritation can be decreased by handling the wet cast with the palms of the hands.

The nurse is coordinating care for a postpartum client and her newborn with the unlicensed assistive personnel (UAP). The mother is human immunodeficiency virus (HIV) positive. The nurse would recognize the need to educate the UAP, who is observed doing which of the following actions? A. Assist the mother who is attempting to breastfeed her baby B. Place the infant on his or her back in the bassinet C. Wear gloves while changing the newborn's soiled diaper D. Assist the mother to walk to the bathroom

A In the United States, it is current practice to counsel a mother who is HIV positive, or has AIDS, against breast feeding; breast feeding can transmit the virus through the breast milk to the infant. It is correct to place an infant on his or her back to prevent sudden infant death syndrome. Standard precautions should be followed when caring for any client; health care providers should wear gloves when they anticipate contact with body secretions (changing a soiled diaper).

An 80 year-old client on digoxin reports nausea, vomiting, abdominal cramps and halo vision. Which laboratory result should the nurse evaluate first? A. Potassium levels B. Blood pH C. Magnesium levels D. Blood urea nitrogen

A Nausea, vomiting, abdominal cramps and halo vision are classic signs of digitalis toxicity. The most common cause of digitalis toxicity is a low potassium level. Clients are to be taught that it is important to have adequate potassium intake, especially if taking loop or thiazide diuretics that enhance the loss of potassium.

The nurse manager uses a block scheduling plan for staffing. However, because staff have asked for many changes and exceptions to the schedule over the past few months, the nurse manager is considering self-scheduling. What type of effect does the nurse manager anticipate with self-scheduling? A. Improved team morale B. Reduced overtime payouts C. Decreased staff turnover D. Improved quality of care

A Nurses in direct care positions are more satisfied when opportunities exist for autonomy and control. The nurse manager becomes the facilitator rather than the decision maker of the schedule for unit needs when self-scheduling exists. Peer pressure and team work are the driving forces during self-schedule approaches.

The client had a plaster cast applied after fracturing the left radius. Two days later, the client reports a constant deep throbbing pain that's unrelieved by narcotics. What should be the first action by the nurse? A. Assess color, temperature and movement of the exposed fingers B. Elevate the left arm a little higher than the level of the heart C. Apply an ice pack to the area of the fracture D. Consider calling the health care provider to ask for a different pain reliever

A Pain that is unrelieved by narcotics, out of proportion to the injury, and described as deep and throbbing, suggests compartment syndrome. The arm must be elevated no higher than the heart to maintain arterial perfusion. Applying ice will do nothing to decreased the tissue pressure in the affected arm. The nurse should first assess the client - remember the 5 Ps: pain, pallor, pulselessness, paresthesia and paralysis. The nurse will then contact the health care provider with the assessment findings. The cast must be bivalved (a longitudinal cut to divide the cast in half) to relieve the pressure and prevent permanent damage to the arm.

The nurse is caring for a client in labor. Which of the following situations will most likely put the neonate at risk for sepsis? A. Premature rupture of membranes B. Cesarean delivery C. Precipitous vaginal birth D. Maternal gestational diabetes

A Premature rupture of the membranes (PROM) is a leading cause of newborn sepsis. After 12 to 24 hours of leaking fluid, actions should be taken to reduce the risk of infection to the mother and the fetus/newborn.

The nurse is planning care for a 6 month-old infant. What must the nurse provide to assist in the development of trust? A. Security B. Food C. Comfort D. Warmth

A Social and emotional development is based on trust, love and security. The best way to help infants develop trust is to give them warm and consistent care so they can form secure attachments. Providing food, comfort and/or warmth are components of care and are required to develop a secure and trusting relationship.

The nurse is preparing a handout on infant feeding to be distributed to families visiting the clinic. Which notation should be included in the teaching materials? A. Solid foods are to be introduced one at a time beginning with cereal B. Egg white is added early to increase protein intake C. Solid foods should be mixed with formula in a bottle D. A variety of ground meat should be started early to provide iron

A Solid foods should be added, one at a time, between 4 to 6 months. If the infant is able to tolerate the food, another is then added each week. Iron-fortified cereal is the recommended first food; rice cereal is recommended due to the low risk of food allergies. Teach parents to mix the cereal flakes with either breast milk or formula. After the baby is eating cereal, pureed meat, vegetables and fruits can be introduced. Egg whites and wheat products should not be given before the baby is at least a year old because these foods are more commonly associated with allergies.

The client is diagnosed with an overactive bladder. The nurse is providing education about a newly prescribed medication, solifenacin succinate. Which statement by the client indicates an understanding of the nurse's teaching? A. "I will need to avoid working in my garden or exercising when it is hot outside." B. "I may experience a rapid weight loss." C. "I understand that I may have loose stools while taking this medication." D. "I may need to drink more fluids to avoid dry mouth syndrome."

A Solifenacin is a cholinergic blocking drug and is used to treat symptoms of overactive bladder, such as frequent or urgent urination and incontinence. Clients should avoid becoming overheated or dehydrated during exercise and in hot weather because it can decrease perspiration and may predispose the client to heat stroke. The medication can also cause blurred vision and constipation. A rarer side effect is a rapid weight gain. While dry mouth is a common side effect, clients should not increase fluid intake; instead, they can either chew sugar-free gum or suck on sugar-free candy.

The nurse has completed discharge teaching to a client who had a total hip arthroplasty. Which statement made by the client indicates further teaching is needed? A. "Now I'll finally be able to bend forward to tie my shoes without pain." B. "When I go home, I should not stand for long periods." C. "If my hip pain gets worse I should call my doctor." D. "I'll use an electric razor to shave."

A Someone who had a total hip replacement should not sit or stand for prolonged periods of time to help prevent thromboembolism and muscle fatigue. Because anticoagulants are typically used postoperatively, the use of an electric razor is indicated. Any increase in hip pain must be evaluated for complications. Following hip replacement surgery, a person should never bend at the waist more than 90 degrees, which would mean the person should not bend over to tie shoes.

A newly admitted client reports taking phenytoin for several months. Which of the following assessments should the nurse be sure to include in the admission report? (Select all that apply.) A. Report of unsteady gait, rash and diplopia B. Report of any seizure activity C. Serum phenytoin levels D. Report of anorexia, numbness and tingling of the extremities

A,B,C Serious adverse outcomes of antiseizure medications such as phenytoin (Dilantin) are unsteady gait, slurred speech, extreme fatigue, blurred vision or feelings of suicide. Increased hunger (not anorexia), increased thirst or increased urination are additional serious side effects.

A client is being prepared for an above-the-knee amputation. Which of the following measures are part of the nurse's responsibilities, which are designed to protect the client? (Select all that apply.) A. Verify any allergies B. Verify that the informed consent form is signed C. Have the client confirm his or her identity, the surgical site and the procedure before administration of any medications D. Verify the surgical leg is marked with indelible marker over, or as close as possible to, the surgical incision site E. Explain the procedure, including any risks, before the client signs the surgical consent form

A,B,C,D Prior to surgery, the nurse can witness the client's signature on the consent form, but explanation of the procedure, including risks and benefits, needs to come from the health care provider. Any allergies must be noted and verified prior to surgery. The surgeon must use an indelible marker on the surgical leg to indicate the incision site; sometimes the nonsurgical leg will be marked with a "NO." In the operating room, a surgical checklist is completed with a nurse and anesthesiologist. Prior to administration of anesthesia, there is another check with the client to verify identify, the correct surgical site and procedure.

The client is being treated for complications of a chronic disease on a medical-surgical unit. Who can have access to the client's medical record? (Select all that apply.) A. The nursing instructor planning clinical assignments B. The facility researcher collecting data for a study to which the client consented C. The certified nursing assistant documenting vital signs D. The emergency department nurse who originally admitted the client and now wants to know the client's current status E. The person who has health care power of attorney F. The client's spouse or other close family member

A,B,C,E Safeguarding client privacy requires strict adherence to the ethical standards of confidentiality and need-to-know access. Only those individuals who are directly involved in the client's care should have access to his or her information. The ED nurse is no longer directly involved in the client's care and should not have access to information about the client. Without valid authorization, such as health care power of attorney, a spouse or other family members cannot access the client's medical records.

The nurse is evaluating an adult client who is receiving continuous enteral nutrition (EN) through a nasogastric tube. Which findings would indicate that the nurse needs to contact the health care provider? (Select all that apply.) A. New onset adventitious lung sounds B. 200 mL dark yellow urine voided in the last eight hours C. Aspirated gastric fluid has a pH of 4 D. Pale and dry oral mucous membranes E. A weight loss of 2 kg in 24 hours F. Gastric residual volume of 100 mL

A,B,D,E Pulmonary aspiration of enteral feeding formula is a risk for clients receiving EN. New onset adventitious or abnormal lung sounds on auscultation in a client receiving EN are indicative of fluid in a lung segment, or narrowing or obstruction of an airway. The health care provider should be notified.Throughout continuous EN, and before intermittent enteral feedings, it is imperative that aspirated secretions be tested for pH levels. In addition, routine aspiration precautions should include strict adherence to standard practices, such as residual volume checks, elevating the head of the bed during feedings and verification of tube exit location markings.Correct placement of the tube in the stomach is verified if the pH is lower than 5. If the tube has been inadvertently placed in a lung or the intestines, the aspirate pH is 6 or higher. Owing to the high risk of adverse events, finding a pH above 6 is a reason to stop the procedure and verify placement through radiologic methods. Aspirate pH may be altered because of medications or feedings (pH value from 4.5-6). If other indicators verify correct placement, waiting and retesting is recommended, rather than radiologic exposure.Delayed gastric emptying is a concern if 250 mL or more remains in a client's stomach (gastric residual volume (GRV)) on two consecutive assessments (one hour apart) or if a single GRV measurement exceeds 500 mL.Due to the nutrient-dense, hypertonic composition of enteral feeding formulas, clients on EN are at risk for developing hyperosmolar dehydration. Signs and symptoms of clinical dehydration include: sudden weight loss (overnight), postural hypotension, tachycardia, thready pulse, dry mucous membranes, poor skin turgor, slow vein filling, flat neck veins when supine and dark yellow urine. If the dehydration is severe, the symptoms will include thirst, restlessness, confusion, hypotension, oliguria (urine output below 30 mL/hr) and cold, clammy skin.

A healthy 18 year-old is entering college in the fall. Which immunization would the health care provider recommend prior to college? (Select all that apply.) A. Seasonal influenza vaccine B. Tetanus, Diphtheria, Pertussis vaccine (Tdap) C. Pneumococcal polysaccharide vaccine (PPSV23) D. Meningococcal conjugate vaccine (MCV4) E. Shingles vaccine F. Human papillomavirus (HPV) vaccine

A,B,D,F Adults older than age 50 should get the shingles vaccine. The PPSV23 is given to adults older than age 65. (The pneumococcal vaccine PCV13 is routinely given to infants/children.) An 18 year-old who is going to college should receive the TDAP, MCV4 and seasonal influenza vaccine. He or she should also receive the HPV vaccine if s/he has not already received it.

The nurse compares the third postoperative assessment findings to the first two postoperative assessments. What action should the nurse take to provide optimal care for this client? (Select all that apply.) 1st Postop Assessment: Blood Pressure; 110/80 mm Hg; Pulse 80; Respiratory Rate 10; Oxygen Saturation 98% 2nd Postop Assessment: Blood Pressure 100/72 mm Hg; Pulse 88; Respiratory Rate 16; Oxygen Saturation 97% 3rd Postop Assessment: Blood Pressure 92/64 mm Hg; Pulse 106; Respiratory Rate 24; Oxygen Saturation 95% A. Elevate the client's lower extremities B. Move the bed into Trendelenburg position C. Assist the client to use the incentive spirometer D. Administer an intravenous fluid bolus E. Inspect the surgical incision site F. Administer pain medication

A,D,E Hypovolemia due to blood loss should be considered in the postoperative client who develops tachycardia and hypotension (a systolic BP reading below 90 in an adult indicates possible shock.) The nurse should check the incision site and any area dependent of the site for any blood loss. Evidence supports elevating the lower extremities in hypotensive episodes, to bring fluid from the lower body to the core; there is no evidence to support using the Trendelenburg position. An IV fluid bolus can also be used to increase volume. Although hypotension and tachycardia may also indicate pain, the nurse should ensure that the client's ABCs are stable before medicating for pain. Assisting the client to use the incentive spirometer can be done later.

The nurse is teaching a client with migraine headaches about almotriptan. Which statement by the client indicates that the teaching was effective? A. "I will wait to take the medication until the pain has become unbearable." B. "I will take the medication as soon as I notice migraine symptoms." C. "If the first dose does not help, I can take two more doses 15 minutes apart." D. "I will take a dose every morning to make sure to prevent an acute attack."

B Almotriptan and other triptans are serotonin receptor agonists that work by causing vasoconstriction of intracranial arteries. The drug is most effective when taken as soon as migraine symptoms start but before the onset of acute pain. It will not prevent headaches or reduce the number of attacks. One of the most common side effects of this medication is dry mouth. After taking a dose, if the headache goes away and comes back, it is acceptable to take a second dose. The client should not take more than two doses of any triptan in 24 hours.

The nurse works in an assisted living facility and cares for older adults. The nurse understands that older adults are at a greater risk for drug toxicity than younger adults due to which physiological change associated with aging? A. Older adults are often malnourished and anemic B. Older adults have less body water and more fat C. Older adults have more rapid hepatic metabolism D. Drugs are absorbed more readily from the gastrointestinal tract

B Because older adults have decreased lean body tissue and water in which to distribute medications, more drug remains in the circulatory system, creating a potential for drug toxicity. Increased body fat results in greater amounts of fat-soluble drugs being absorbed, leaving less medication in circulation, thus increasing the duration of action of the drug.

The charge nurse in a long-term care (LTC) facility is making assignments for staff. Which assignment is appropriate for the certified nursing assistant (CNA)? A. Teach family members how to perform passive range-of-motion exercises B. Calculate and record intake and output C. Apply a dry dressing to a skin tear D. Provide oral suctioning as needed for an unresponsive client

B CNAs are trained to perform a number of tasks or basic nursing skills, including calculating and recording intake and output. Although CNAs can wash and apply emollients on skin, they should not apply dressings. CNAs in LTC would not suction a client's mouth. Although CNAs can perform passive range-of-motion exercises, they cannot teach others how to do this.

The nurse is preparing to administer medications for a client with a gastrostomy tube. The nurse should contact the health care provider before giving which drug through the gastrostomy tube? A. Digoxin (Lanoxin) liquid B. Diltazem SR (Cardizem SR) tablet C. Acetaminophen (Tylenol) liquid D. Calcium carbonate (Os-cal) tablet

B Cardizem SR is a "sustained-release" drug form. Sustained release, controlled release or long-acting drug formulations are designed to release the drug over an extended period of time. If crushed, as would be required for gastrostomy tube administration, sustained-release properties and blood levels of the drug will be altered. The health care provider must substitute another medication.

A client admits to benzodiazepine dependence for over the past several years. The client is now in an outpatient detoxification program. The nurse should understand that a priority during withdrawal from any substance is which of these actions by the client? A. Expect mild physical symptoms B. Avoid alcohol use during this time C. Discontinue the drug by weaning D. Rise slowly from a lying to standing position

B Central nervous system depressants interact with alcohol. The client will gradually reduce the dosage under the health care provider's direction. During this time, alcohol must be avoided. The other options are correct. However, the question asks for a priority, which is the correct answer.

An 82-year-old male client is admitted with benign prostatic hyperplasia (BPH). Which finding by the nurse will require immediate action? A. A blood pressure of 180/105 B. A bladder ultrasound value of 900 mL C. A heart rate of 110 bpm D. Severe abdominal pain

B Complications of BPH include acute urinary retention. Urinary retention is the accumulation of urine in the bladder due to bladder outlet obstruction caused by the enlarged prostate gland. Acute urinary retention is a medical emergency that requires prompt bladder drainage.The elevated heart rate and blood pressure and the severe abdominal pain are signs and symptoms of the acute retention. They will most likely resolve when the retention is resolved. The high bladder scan/ultrasound value confirms the retention of a large volume of urine that will require catheterization.

A mother asks a nurse if she should be concerned about her child's tendency to stutter. What assessment data will be most useful in counseling the parent? A. Stressful family events B. Age of the child C. Sibling position in family D. Parental discipline strategies

B During the preschool period children use their rapidly growing vocabulary faster than they can produce their words. This failure to master sensorimotor integrations results in stuttering. This dysfluency in speech pattern is a normal characteristic of language development. Therefore, knowing the child's age is most important in determining if any true dysfunction might be occurring with stuttering.

A partner is concerned because the client frequently daydreams about moving to Arizona to get away from the pollution and crowding in southern California. What approach should the nurse use in an explanation? A. Detaching or dissociating in this way postpones painful feelings B. Such fantasies can gratify unconscious wishes or prepare for anticipated future events C. Converting or transferring a mental conflict to a physical symptom can lead to conflict within the partnership D. Isolating the feelings in this way reduces conflict within the client and with others

B Fantasy is imagined events (daydreaming) to express unconscious conflicts or gratify unconscious wishes. The other options cannot be applied to this situation.

A nurse is assessing a 6 year-old child for the first time in the clinic and finds that the child has deformities of the joints, limbs and fingers; a thinned upper lip; and small teeth with faulty enamel. The mother states: "My child seems to have problems in learning to count and recognizing basic colors." Based on this data, the nurse suspects that the child is most likely showing the effects of which problem? A. Chronic toxoplasmosis B. Fetal alcohol syndrome (FAS) C. Congenital abnormalities D. Lead poisoning

B Major features of fetal alcohol syndrome (FAS) are facial and other malformed physical features, such as small head circumference and brain size (microcephaly), small eyelid openings, a sunken nasal bridge, an exceptionally thin upper lip, a short, upturned nose and a smooth skin surface between the nose and upper lip. Vision difficulties include nearsightedness (myopia). Other findings are mental retardation, delayed development, abnormal behavior such as short attention span, hyperactivity, poor impulse control, extreme nervousness and anxiety. Behavioral problems, cognitive impairment and psychosocial deficits are also associated with this syndrome.

A postoperative client is admitted to the post anesthesia care unit (PACU). An anesthetist reports that malignant hyperthermia occurred during surgery. A nurse should approach the care of this client with what knowledge about this complication? A. A pre-existing bacterial infection precipitated the situation B. A genetic predisposition acts as the stimulus to such a reaction C. It is an allergic response to general anesthesia D. Selected surgical procedures place clients at a higher risk for this complication

B Malignant hyperthermia is a rare, potentially fatal adverse reaction to inhaled anesthetics. There is a genetic predisposition to this disorder. Findings include: a rapid rise in temperature to 105 F (40.5 C) or higher, muscle rigidity and stiffness, dark brown urine and muscle aches without a history of obvious exercise to explain sore muscles.

An 85 year-old client reports experiencing generalized muscle aches and pains. What should be the first action by a nurse? A. Reassure the client that this is not unusual for age B. Assess the severity and location of the pain C. Encourage the client to gradually increase daily activity D. Request an order for a nonsteroidal anti-inflammatory drug

B Most older adults have one or more chronic painful illnesses, and in fact, they often must be asked about discomfort (rather than "pain") to reveal the presence of pain. There is no evidence that pain of older adults is less intense than in younger adults. It is important for the nurse to assess the pain thoroughly before the implementation of pain relief measures or recommendations. The nurse should also be sure to ask clients about any drugs they are taking (OTC, prescribed and herbal); certain medications, such as the statins, can cause muscle aches in the legs.

A nurse is teaching a smoking cessation class and notices there are two pregnant women in the group. Which information is a priority for these women? A. The placenta serves as a barrier to nicotine B. There is a relationship between smoking and low birth weight C. Moderate smoking is effective in weight control D. Low tar cigarettes are less harmful during pregnancy

B Nicotine reduces placental blood flow and may contribute to fetal hypoxia or placenta previa, which results in a decreased growth potential of the fetus.

When walking past a client's room, the nurse hears an unlicensed assistive person (UAP) talking to another UAP. Which of these statements requires further intervention by the nurse? A. "I'll come back and make the bed after I go to the lab." B. "Since I am late for lunch, would you perform my client's blood glucose test?" C. "This client seems confused, we need to watch the client closely." D. "If we work together we can get all of the client care completed."

B Only registered nurses (RNs) and licensed practical or vocational nurses (LPN/VNs) can assign tasks and activities. UAPs cannot re-assign tasks or activities to other UAPs. Nurses are accountable for all nursing care; if UAPs cannot complete assignments, they should notify the nurse, who will reassign the task.

A child is to have chest physiotherapy (CPT) by the nurse. Which nursing action is appropriate? A. Schedule the therapy 30 minutes after meals B. Confine the percussion to the rib cage area C. Teach the child not to cough during the treatment D. Place the child in a prone position for the therapy

B Percussion (clapping) should be done in the area of the rib cage anterior and posteriorly. The position depends on the desired outcome for secretion removal. This therapy should be done one hour prior or two hours after meals.

The nurse is providing postoperative care for a client who has undergone a laparoscopic cholecystectomy. Which assessment finding should be reported immediately to the health care provider? A. Client reports shoulder pain B. Client reports severe right upper quadrant tenderness C. Client is drowsy D. Absence of bowel sounds

B Shoulder pain is a common complaint following laparoscopic surgery due to the effects of carbon dioxide gas. Postoperative drowsiness is expected. Although bowel sounds should be assessed after surgery, absence of bowel sounds immediately after surgery is not a cause for alarm. Right upper quadrant pain could be from a retained gallstone or bile duct injury; severe postoperative pain in the right upper quadrant is a medical emergency after a laparoscopic cholecystectomy.

The unlicensed assistive person (UAP) reports to the registered nurse (RN) that the client has a blood pressure of 78/46 mm Hg and a pulse of 116 BPM using electronic equipment. Which action should the nurse implement first? A. Notify the health care provider immediately B. Assess the client's cardiovascular status C. Place the client in Trendelenburg position D. Instruct the UAP to recheck the vital signs manually and report back with the information

B The RN is responsible for client care and must complete an assessment in order to plan for and provide appropriate interventions. It's not necessary to notify the health care provider until the report of the vital signs has been verified (by using a different machine or manually taking the pulse), an assessment by the RN has been completed, and the data compared with previously recorded vital signs. Even though the Trendelenburg position is still taught in schools, recent evidence does not support the use of this technique; even so, the nurse must still assess the client before taking further action.

The nurse is performing triage in a hospital's emergency department. Which of these clients would the nurse identify as most critical and being first in need of intervention? A. An adolescent who has soot over the face and shirt B. A 5 month-old infant who has audible wheezing and grunting C. A toddler with singed ends of long hair that extends down to the waist D. A middle-aged man with second-degree burns over the right hand

B The age and the findings suggest this client is at immediate risk for respiratory complications. The other clients are at lesser risk for respiratory problems.

A nurse admits a 50 year-old client with a three-day history of swelling of the face, hands and feet; foamy brown urine; fever and malaise. Which information obtained in the admission interview alerts the nurse that these findings may reflect a diagnosis of acute glomerulonephritis? A. History of mild hypertension B. Travel to a foreign country C. Sore throat two weeks ago D. Type 1 diabetes since age 15

C Glomerulonephritis commonly presents with proteinuria (foamy urine) that is rusty or brownish in appearance and swelling due to the systemic protein loss. In the majority of cases of acute glomerulonephritis, there is a history of an untreated streptococcal throat infection preceding the onset of symptoms by two to three weeks. The other options are not directly related to the development of acute glomerulonephritis.

A client had a thoracotomy with a right upper lobectomy. The nurse should focus on pain management for which of the following reasons postoperatively? A. Maintain full range of motion B. Relaxation and sleep C. Deep breathing and coughing D. Internal incisional healing

C A lobectomy is often performed to treat tuberculosis, bronchiectasis and cancer. Postoperatively, the focus of care is to prevent respiratory complications, such as atelectasis and pneumonia. Without proper pain management, clients will be reluctant to cough and deep breathe, which will predispose them to these and other complications.

The nurse is working in an inpatient psychiatric unit. Which statement made by a client indicates that the client may have a thought disorder? A. "I'm fine. It's my daughter who has the problem." B. "I'm a little confused. What time is it?" C. "I can't find my 'mesmer' shoes. Have you seen them?" D. "I'm so angry about this. Wait until my partner hears about this."

C A neologism is a word that is self-invented by a person and not readily understood by another person. The use of neologisms is often associated with a thought disorder. The other statements reflect appropriate connections between the expressed thoughts. Thought disorders are associated with schizophrenia, delusions and hallucinations of psychosis.

The client has decreased adrenal function. What should the nursing care plan for this client include? A. Place the client in reverse isolation B. Prevent constipation C. Limit the number of visitors D. Encourage physical activity

C Any stress, either physical or emotional, places additional stress on the adrenal glands, which could precipitate an Addisonian crisis in this client. The plan of care should protect the client from stress by avoiding the emotional stress of (too many) visitors and by reducing physical activity until the client's condition stabilizes.

After a heart attack, the health care provider orders low-dose (81 mg) aspirin at bedtime for the client. The client wants to know why a "baby aspirin" was prescribed and not a 325 mg tablet. What's the best reason why the client should not use the higher dose of aspirin? A. It will affect the client's hearing B. It will cause heartburn C. It will increase the risk of bleeding D. It will interfere with normal sleep patterns

C Aspirin is a nonsteroidal anti-inflammatory drug and is prescribed to help keep blood clots from forming after a heart attack. Lower-dose aspirin therapy is just as effective in reducing the risk of secondary heart attacks as higher doses of aspirin, but with less risk of bleeding (including gastrointestinal bleeding.) This is especially important for the client to understand since he will may also be prescribed an anticoagulant after his heart attack. Common side effects of aspirin therapy include rash, upset stomach, heartburn, drowsiness, and headache. Many drugs, including aspirin, can affect hearing; usually much larger daily doses would be needed to affect hearing.

A nurse is beginning nutritional counseling/teaching with a pregnant woman. What is the initial step in this interaction? A. Question her understanding and use of the food pyramid B. Explain the changes in diet necessary for pregnant women C. Conduct a diet history to determine her normal eating routines D. Teach her how to meet the needs of self and her family

C Assessment is always the first step in planning teaching for any client. A thorough and accurate history is essential for gathering the needed information. The results of this information provides the basis of the planned educational needs.

A nurse is planning to give a 3 year-old child oral digoxin. Which action is the best approach by the nurse? A. "This is your medicine, and you must take it all right now." B. "Do you want to take this pretty red medicine?" C. "Would you like to take your medicine from a spoon or a cup?" D. "You will feel better if you take your medicine."

C At 3 years of age a child often feels a loss of control when hospitalized. Giving a choice about how to take the medicine allows the child to express an opinion and have some control.

A nurse provides instructions to a new mother on the proper techniques for breast-feeding her infant. Which statement by the mother is incorrect and indicates a need for additional instruction? A. "There may be times that I will need to manually express milk." B. "The baby should latch onto the nipple and areola areas." C. "I can switch to a bottle if I need to take a break from breast-feeding." D. "I should position my baby completely facing me with my baby's mouth in front of my nipple."

C Babies adapt more quickly to the breast when they are not confused about what is put into their mouths and its purpose. Artificial nipples do not lengthen and compress the way the human nipples (areola) do. The use of an artificial nipple weakens the baby's suck as the baby decreases the sucking pressure to slow fluid flow. Babies should not be given a bottle during the learning stage of breast-feeding.

The client has an order for benztropine. After assessing the client, which condition would contraindicate the use of this medication? A. Neuromalignant syndrome B. Acute extrapyramidal syndrome C. Glaucoma D. Parkinson's disease

C Benztropine is an anticholinergic medication used to treat extrapyramidal disorders caused by antipsychotic medications or Parkinson's disease. Use of benztropine or another anticholinergic is contraindicated for individuals diagnosed with glaucoma, ileus and prostatic hypertrophy. Adverse effects include tachycardia, anticholinergic psychosis and heat stroke.

A client is receiving digoxin 0.25 mg by mouth daily. A health care provider has written a new order to give metoprolol tartrate 25 mg twice a day by mouth. In assessing the client prior to administering the medications, which finding should the nurse report to the health care provider? A. Urine output of 50 mL/hour B. Respiratory rate of 16 C. Blood pressure of 94/60 D. Heart rate of 76 BPM

C Both medications decrease the heart rate. Metoprolol (Lopressor) affects blood pressure. Therefore, the heart rate and blood pressure must be within normal range (HR 60 to 100 BPM and systolic BP greater than 100 mm Hg) in order to safely administer both medications.

A nurse admits a 3 week-old infant to the special care nursery with a diagnosis of bronchopulmonary dysplasia. As the nurse reviews the birth history, which data would be most consistent with this diagnosis? A. Phototherapy was used to treat Rh incompatibility B. Meconium was cleared from the airway at delivery C. The infant received mechanical ventilation for two weeks D. Gestational age assessment suggested growth retardation

C Bronchopulmonary dysplasia (BPD) is an iatrogenic disease caused by mechanical ventilation. When the prematurely born infant is treated with mechanical ventilation, over time the pressure from the ventilation and excess oxygen can injure the infant's lungs, causing BPD.

A client is receiving external beam radiation to the mediastinum for treatment of bronchial cancer. Which of these problems should be addressed as a priority in planning care? A. Esophagitis B. Skin irritation C. Leukopenia D. Fatigue

C Clients being treated by radiation over the sternum, which is a bone marrow producing area, develop leukopenia due to the depressant effect of radiation therapy on the bone marrow function. With the resultant low white counts, infection is a potential outcome. The other options are possible complication outcomes of radiation therapy on this part of the body. However, they are not the priority because leukopenia is a threat to the entire body and the other options are more of a local problem.

The parent of a 2 year-old reports the child has experienced mild diarrhea for the past two days. Which statement by the nurse provides the best nutritional information for the child? A. Clear liquids and gelatin for 24 hours B. NPO for 24 hours, then rehydrate with milk and water C. Continue with the regular diet and include oral rehydration fluids D. Offer bananas, apples, rice and toast as tolerated

C Current recommendations for mild to moderate diarrhea are to maintain a normal diet with fluids to rehydrate. If the diarrhea was severe then the BRAT (for bananas, apples, rice and toast) diet may be appropriate.

A nurse is caring for a client with active tuberculosis who has a history of noncompliance. Which of these actions by the nurse would represent appropriate care for this client? A. Ask the health care provider to change the regimen to fewer medications B. Schedule weekly clinic visits for the client C. Ask a family member to supervise daily compliance D. Instruct the client to wear a high efficiency particulate air mask in public places

C Direct-observed therapy (DOT) is a recognized method for ensuring clients' compliance to drug regimens. A program can be set up to directly observe the client taking the medication in the clinic, home, workplace or other convenient location.

The nurse is caring for a mother who has just delivered a stillborn infant. What would be the most therapeutic nursing intervention? A. Reassuring the mother that she now has an angel in heaven watching over her. B. Explaining to the mother that she is young and will have other children. C. Offering the mother the opportunity to hold, bathe and dress the infant. D. Notifying the hospital chaplain to come and pray with the mother.

C The loss of an infant has special meaning for grieving parents. To help them understand that the death is a reality and to facilitate their grieving, it is important to offer the opportunity to hold the infant while dying or after the delivery and to provide a quiet, private place for the parents with their child. Allow the parents to have as much time with their child as they request. Differences in gender, cultural practices and religious beliefs will affect the parents' grief response and the nurse needs to be alert for verbal and nonverbal cues.Giving false reassurance, pointing out that future pregnancies are possible and arranging for a chaplain's visit are nontherapeutic interventions that are insensitive to the mother's current, emotional needs and are based on cultural bias and assumptions.

The client who is experiencing an acute asthmatic episode is admitted to the hospital with intermittent nonproductive coughing. The pulse oximeter reading is 88%. The client states to the nurse, "I feel like this is going to be a bad episode. I wish I would not have gone into that smoky bar last night." Which nursing diagnosis would be the priority for this client? A. Anxiety related to hospitalization B. Altered health maintenance related to preventative behaviors associated with asthma C. Impaired gas exchange related to bronchoconstriction and mucosal edema D. Ineffective airway clearance related to potential thick secretions

C The low pulse oximetry reading indicates poor gas exchange, with inadequate oxygenation of arterial blood resulting from the bronchoconstriction and mucosal edema of the acute asthma attack. Treatments will include oxygen administration to correct the hypoxia, titrated to maintain a saturation at or above 92-95%, intravenous or oral corticosteroids to reduce the mucosal edema and inhaled bronchodilators such as albuterol via nebulizer or metered dose inhaler with a spacer to treat airway bronchoconstriction. While the other diagnoses may be appropriate for this client, they are not the priority at this time.

The nurse is teaching a client who has a new prescription for sublingual nitroglycerin. Which point should the nurse emphasize? A. Take the medication at the same time each day B. Rest in bed for an hour after taking medication C. Carry the nitroglycerine with you at all times D. Keep the medication bottle in the refrigerator

C The medication should be kept in its original dark-colored glass container. Nitroglycerin should be carried by the client at all times so it can be used when anginal pain occurs. When needed, the client should sit and place tablet under his or her tongue. Sitting is safe because the drug can cause lightheadedness or dizziness, but it's not necessary to rest in bed. The client should never pack this and any other medications in a checked a bag when traveling.

A newly admitted client reports gaining 5 pounds (2.27 kg) the past week even though he has hasn't been very hungry. The nurse observes swelling of the feet and ankles. What is the most likely explanation for the weight gain? A. Hyperthyroidism B. Malnutrition C. Congestive heart failure D. Acromegaly

C The unexplained rapid weight gain is probably due to fluid retention. Clients who gain as little as two pounds (0.9 kg) in a week may require hospitalization due to worsening heart failure. The lack of appetite (or a feeling of being full) and edema are also signs of worsening heart failure. Hypothyroidism, and not hyperthyroidism, can lead to low body temperature, which causes fluid retention or bloating. Low protein levels in the blood caused by malnutrition can cause edema. However, there's not enough information given in the question to know if this client is malnourished or not. Acromegaly is characterized by overgrowth of body tissues, not edema, and is caused by excessive secretion of growth hormone.

The public health nurse is reviewing data about trends in tuberculosis (TB) in the U.S. Which factor should the nurse understand about the current trends in TB? A. There is no significant difference between TB rates for foreign-born and U.S.-born people B. NonHispanic Asians have the lowest TB case rate C. All TB clients should be counseled and tested for HIV D. The number of new cases of TB in the U.S. continues to increase each year

C There has been a steady decrease in the total number of new cases of TB in the U.S. over the last nine years. According to current statistics, non-Hispanic Asians have the highest TB case rate in the U.S. and more than half of all new TB cases were in foreign-born people. The American Thoracic Society and the Infectious Disease Society of American recommend that everyone who tests positive for TB be counseled and tested for HIV; conversely, people with HIV or AIDS should be tested for TB because the chance of having both diseases is extremely high.

A client is admitted with a pressure ulcer in the sacral area. The partial thickness wound is 1.5 x 2.7 inches (4 cm x 7 cm), the wound base is red and moist with no exudate and the surrounding skin is intact. Which covering should the nurse select for this wound? A. Wet to dry dressing B. Dry sterile dressing with antibiotic ointment C. Occlusive moist dressing D. Transparent dressing

C This wound has granulation tissue present and must be protected. The granulation tissue is evident by wound base being red and moist with no exudate. The use of a moisture retentive dressing is the best choice because moisture supports wound healing.

A nurse is reinforcing teaching to a 24 year-old woman receiving acyclovir for a herpes simplex virus type 2 infection. Which of these instructions should the nurse give the client? A. Continue to take prophylactic doses for at least five years after the diagnosis B. Complete the entire course of the medication for an effective cure C. Begin treatment with acyclovir at the onset of symptoms of recurrence D. Stop treatment if she thinks she may be pregnant to prevent birth defects

C When the client is aware of early symptoms, such as pain, itching or tingling, treatment is very effective. Medications for herpes simplex do not cure the disease. They simply decrease the intensity of the symptoms. Acyclovir (Zovirax) is not known to have an impact on the fetus. Acyclovir should not be taken for preventive purposes, regardless of the date of diagnosis.

The nurse is teaching a parent about side effects of routine immunizations. Which of these findings must be reported immediately? A. Irritability B. Local tenderness C. Seizure activity D. Slight edema at site

C While severe complications are rare, any seizure activity must be immediately reported; seizures can occur up to 7 days after injection. Other reactions that should be reported include crying for more than three hours, temperature over 105 F (40.5 C) following DTaP immunization, and tender, swollen, reddened areas where the shot was given.

A newly appointed nurse manager is having difficulties with time management. Which advice from an experienced manager should the new manager implement first? A. Set daily goals and establish priorities for each hour and each day B. Complete each task before beginning another activity in selected instances C. Ask for additional assistance when you feel overwhelmed D. Keep a time log of your day in hourly blocks for at least one week

D Apply the nursing process to time management, so the assessment of the current activities is the initial step. A baseline is established for activities and time use so that needed changes can be pinpointed.

A nurse is assigned to care for a client who had a myocardial infarction (MI) two days ago. The client has many questions about this condition. What area is a priority for the nurse to discuss with the client at this time? A. The overview cardiac rehabilitation B. Activity and rest guidelines C. Medication and diet guideline D. Daily physical and mental needs and concerns

D At two days after MI, the client's education should be focused on the immediate needs and concerns for the day. The physical needs are an initial focus. Then the mental or psychosocial needs are secondary.

A client who had a vasectomy is in the post anesthesia care unit (PACU) at an outpatient clinic. Which of these points is most important to be reinforced by the nurse? A. "The health care provider at this clinic recommends rest, ice, an athletic supporter or over-the-counter pain medication to relieve any discomfort." B. "After your vasectomy, strenuous activity needs to be avoided for at least 48 hours. If your work doesn't involve hard physical labor, you can return to your job as soon as you feel up to it. The stitches generally dissolve in 7 to 10 days." C. "This procedure doesn't impede the production of male hormones or the production of sperm in the testicles. The sperm can no longer enter your semen and no sperm are in your ejaculate." D. "Until the health care provider has determined that your ejaculate doesn't contain sperm, continue to use another form of contraception."

D All of these options are correct information. The most important point to reinforce is the continued need to take additional action for birth control until it is determined that no risk is present for a possible pregnancy outcome.

A 76 year-old client who smokes one pack of cigarettes per day is diagnosed with chronic obstructive pulmonary disease (COPD). The nurse is teaching the client and family members about the use of oxygen by nasal cannula in the home. Which information is most important for the nurse to include in the discharge instructions? A. Adjust the liter flow to 5 L as needed for shortness of breath B. Turn off oxygen during meals C. The client should not smoke, now that he has been diagnosed with COPD D. The client should not smoke while wearing oxygen

D Because oxygen supports combustion, there is a risk of fire if anyone smokes near the oxygen equipment. The client should take off the oxygen, turn off the flow meter and go to another part of the home or outside to smoke. While continuing to smoke will cause the client's COPD to get worse, it can be very challenging for people who have smoked for many years to quit. Smoking cessation should be encouraged and supported in ways that are appropriate to the client's readiness to cut down or quit, but the most important teaching point at discharge is to stress not smoking while wearing or near the oxygen supply.

During the two-month well-baby visit, the mother explains that formula seems to stick to her baby's mouth and tongue. Which assessment would provide the most valuable data for a nurse? A. Obtain cultures of the mucous membranes B. Inspect the baby's mouth and throat C. Flush both sides of the mouth with normal saline D. Use a soft cloth to attempt to remove the patches

D Candidiasis can be distinguished from coagulated milk when attempts to remove the patches with a soft cloth are unsuccessful or trigger bleeding of the tongue under the white substance.

The nurse is caring for a client taking antipsychotic drugs. Why is it important for the nurse to monitor blood pressure in this client? A. Rising trends in blood pressure will indicate when an antiparkinsonian drug is needed B. Most antipsychotic drugs cause elevated blood pressure C. Blood pressure will determine if dietary restriction of sodium is needed D. Orthostatic hypotension is a common side effect

D Clients should be made aware of the possibility of dizziness and syncope from postural hypotension for about an hour after taking an antipsychotic medication. Clients should be advised to get up slowly from a sitting or lying position.

A client who is terminally ill has been receiving high doses of an opioid analgesic for the past month. As death approaches and the client becomes unresponsive to verbal stimuli. What orders should the nurse expect from the health care provider? A. Discontinue the analgesic B. Prescribe a less potent drug C. Decrease the analgesic dosage by half D. Continue the same analgesic dosage

D Clients who are actively dying and have been experiencing chronic pain, will probably continue to experience pain even though they cannot communicate this. Pain medication should be continued at the same dose as long as it is effective at that dose; some adjustment may be needed based on the client's physical manifestations of pain, such as grimacing or moaning.

The nurse is caring for a postoperative client who had a laparotomy. Which of these actions is the most effective nursing intervention to prevent atelectasis from developing? A. Ambulate client within 12 hours postop B. Splint the incision C. Maintain adequate hydration D. Assist the client to slowly deep breathe and cough

D Deep air excursion by slow deep breathing and coughing will expand the lungs and stimulate surfactant production. This is the priority to prevent pulmonary complications after surgery. The nurse should instruct the client on how to splint the abdomen when coughing. Humidification, hydration and nutrition all play a part in the prevention of atelectasis.

The emergency department is sending a client with a diagnosis of delirium tremens to the floor. The admitting nurse would expect which of the following findings? A. A generalized shaking of the body accompanied by repetitive thoughts expressed verbally B. Single or multiple jerks caused by rapid contracting muscles with alternating relaxation C. Disorganized thinking, feelings of terror and non-purposeful behavior D. An excited state accompanied by disorientation, hallucinations and tachycardia

D Delirium tremens is a severe form of alcohol withdrawal that involves sudden and severe changes of the nervous system. The client initially presents with impaired cognition or agitation. The other findings typically progress in the following order: confusion, disorientation, agitation, hallucinations, diaphoresis, fever, tachycardia, hypertension and extreme tremors.

A nurse is evaluating the growth and development of a toddler with AIDS. The nurse would anticipate which finding in this child? A. Delay in musculoskeletal development B. Achieve developmental milestones at an erratic rate C. Difficulty with speech development D. Delay in achievement of most developmental milestones

D Developmental delays are common in children with AIDS, and after achievement of normal development, there may be loss of milestones. The majority of children with AIDS have neurological involvement. There is decreased brain growth as evidenced by microcephaly and abnormal neurologic findings. The other options are accurate but are too limited to be the best response.

A 23 year-old single client in the 33rd week of her first pregnancy tells the nurse that she has everything ready for the baby and has made plans for the first weeks together at home. Which normal emotional reaction does the nurse recognize? A. Focus on fetal development B. Acceptance of the pregnancy C. Ambivalence about pregnancy D. Anticipation of the birth

D Directing activities toward preparation for the newborn's needs and personal adjustment are indicators of appropriate emotional response in the third trimester and a part of "nesting" according to Rubin. Ambivalence about pregnancy is an expected emotion during the first trimester. Acceptance of the pregnancy with a focus on fetal development are important in the second trimester.

A nurse detects fluid leaking from the nose and ears of a client admitted for head trauma. Which is the appropriate nursing action? A. Pack the nose and ears with sterile gauze B. Raise the head of the bed so the client is sitting upright C. Position an ice pack on the back of the neck D. Place loose dressings below the nostril and ear

D Drainage from the nose and ear in a client with head trauma may be caused by leakage of cerebrospinal fluid (CSF). Applying a loose dressing to the nose and ears permits the fluid to drain and provides a visual reference to assess the type and amount of drainage. The nose and ear should not be packed if CSF leakage is suspected. If the drainage contains blood, look for the halo sign: a ring formed when protein in the CSF migrates to the edge of the wet spot as the gauze dries. If the drainage is clear, the nurse should check it for glucose. The head of the bed should be elevated 15 to 30 degrees to facilitate the drainage and decrease intracranial pressure.

A 5 year-old child is rushed to the emergency department approximately six hours after ingesting an undetermined amount of acetaminophen. Which lab test should receive priority attention by the nurse? A. Alanine transaminase test (ALT) and aspartate transaminase test (AST) B. Prothrombin time (PT) and INR C. Electrolytes and blood urea nitrogen (BUN) D. Serum acetaminophen concentration (APAP)

D Emergency treatment of acetaminophen overdose involves checking the client's 4-, 6-, and 8-hour acetaminophen concentration (APAP) levels. These levels will determine N-acetylcysteine (NAC) therapy (the antidote). Clients who ingest an acute overdose and have NAC therapy initiated within 8 hours usually do well and do not develop liver failure. However, because acetaminophen poisoning can lead to liver failure, it is important to evaluate hepatotoxicity; ALT and AST will help determine the degree of liver cell damage. PT (or INR) may also be used to detect impaired liver function. BUN and electrolytes may show renal impairment or acidosis.

Which of these statements best describes the characteristic of an effective reward-feedback system? A. Staff are given feedback in equal amounts over time B. Positive statements should precede a negative statement C. Performance goals should be higher than what is attainable D. Specific feedback is given as close to the event as possible

D Feedback is most useful when given immediately. Positive behavior is strengthened through immediate feedback, and it is easier to modify problem behaviors if what constitutes appropriate behavior is clearly understood.

A nurse is providing instructions for a client with asthma who is allergic to house-dust mites. Which information about prevention of asthma episodes would be the most helpful to include during the teaching? A. Wash bed linens in warm water with a cold rinse B. Open the curtains to let the sunlight in each morning C. Change the pillow covers every month D. Wash and rinse the bed linens in hot water

D For asthma clients who are allergic to house-dust mites, the mattresses and pillows should be encased in allergen-impermeable covers. All bed linens such as pillow cases, sheets and blankets should be washed and rinsed weekly in hot water at temperatures above 130 F (54.4 C), the temperature necessary to kill the dust mites.

A client is newly diagnosed with bipolar disorder and has a prescription for lithium. Which of these points should the nurse be sure to emphasize? A. Substitute generic form if desired B. Maintain a salt-restricted diet C. Take other medication as usual D. Report vomiting or diarrhea

D If dehydration results from vomiting, diarrhea or excessive perspiration, the client may experience findings of toxicity due to a build up of the drug. Lithium has a relatively narrow therapeutic index. Clients with serum lithium levels higher than 2 mEq/L should be admitted to the hospital.

A client is diagnosed with iron-deficiency anemia. What is the cause of the symptoms associated with this condition? A. Decreased cardiac output B. Reduced oxygen saturation C. Destruction of red blood cells (RBCs) D. Tissue hypoxia

D Iron-deficiency anemia is when the RBCs are unusually small and pale (due to their low hemoglobin content.) Tissue hypoxia is the result of not having enough functioning hemoglobin in the blood to oxygenate the tissues; tissue hypoxia is responsible for symptoms such as fatigue, leg cramps and chewing ice. It's possible the client could have a normal SpO2 because the small amount of hemoglobin that's in the blood may be well saturated with oxygen. Anemia does not decrease cardiac output. Destruction of RBCs is associated with hemolytic anemia, not iron-deficiency anemia.

The nurse is assessing a client during a visit to a community mental health center. The client discloses that "I have been thinking about ending my life." Which statement would be the nurse's BEST response to this information? A. "We will help you deal with those thoughts." B. "Do you want to discuss this with your pastor?" C. "Is your life so terrible that you want to end it?" D. "Have you thought about how you would do it?"

D Most experts believe that people who commit suicide don't want to die; they just want to stop hurting. When a client tells you s/he is thinking about death or suicide, you must evaluate the immediate danger the person is in. The correct option provides an opening to discuss the plan, the means (pills, gun, etc.), time set for doing it, and intent to commit suicide. Clients who have formulated a suicide plan are closer to suicidal behavior than those who have vague, nonspecific thoughts.

A postpartum mother is unwilling to allow the father to participate in the newborn's care, although he is interested in doing so. She states, "I am afraid the baby will be confused about who the mother is. Baby raising is for mothers, not fathers." The nurse's initial intervention should be what focus? A. Arrange for the parents to attend infant care classes B. Talk with the father and help him accept the wife's decision C. Discuss with the mother sharing parenting responsibilities D. Set time aside to get the mother to express her feelings and concerns

D Nonjudgmental support for expressed feelings may lead to resolution of competitive feelings in a new family. Cultural influences may also be clarified.

A nurse is giving instructions to the parents of a child with cystic fibrosis. What information should the nurse emphasize about administration of pancreatic enzymes? A. Each time a high-carbohydrate or high-fat meal is eaten B. Crush the tablet and sprinkle on food three times a day C. Once each day in the morning D. With each meal or snack

D Pancreatic enzymes are necessary for digesting fat, starch and protein. They should be taken with each meal and most snacks to allow for the proper digestion of the food. If taken on an empty stomach, they may cause gastric irritation and possibly ulcers. Enzyme capsules should be swallowed whole, not crushed or chewed, and the microspheres should not be sprinkled on or mixed with the whole meal.

A client with cirrhosis of the liver underwent a paracentesis yesterday. Today, the unlicensed assistive personnel (UAP) reports the client is lethargic and has musty-smelling breath. Which assessment should the nurse perform next? A. Auscultate the upper abdomen for bruits B. Monitor the client's clotting status C. Measure the abdominal girth changes D. Assess for flap-like tremors of the hands

D Subtle changes in mental status and a musty odor to the breath are findings associated with hepatic encephalopathy. Hepatic encephalopathy is often seen in people with chronic liver disease (cirrhosis or hepatitis). A classic sign of this disorder is flapping tremors of the hands (asterixis).

A licensed practice nurse (LPN) from the float pool is sent to an adult medical-surgical unit. With this newly added staff person, the charge nurse needs to revise assignments for the shift. Which of the following clients are appropriate to assign to the float pool LPN? A. An older client with newly diagnosed type 2 diabetes and a client who is HIV-positive with a diagnosis of pneumonia B. A trauma victim newly admitted with a diagnosis of quadriplegia and a client one day postoperative radical neck dissection C. A young adult client with a history of schizophrenia and with current alcohol withdrawal syndrome and a client diagnosed with chronic renal failure and anemia D. A middle-aged client diagnosed with hemiplegia and with a gastrostomy tube and a client with a below-the-knee amputation (BKA) who will be starting physical therapy

D The client diagnosed with hemiplegia (and a gastrostomy tube) and the client who is starting physical therapy following a BKA require supportive care and interventions that are within the scope of practice of a LPN. These clients are the most stable and have a minimal risk of complications. The clients in the other options require RN care. Some of the clues are: "newly admitted," "newly diagnosed" and "current alcohol withdrawal" - each of these clients have a high risk of instability and/or require the specialized nursing knowledge, skill or judgment of a registered nurse.

A 72 year-old client diagnosed with osteomyelitis requires a six-week course of intravenous antibiotics. In planning for home care, what is the priority approach by a nurse? A. Investigate the client's insurance coverage for home IV antibiotic therapy B. Select the appropriate venous access device for the long-term IV medication C. Determine if there are adequate handwashing facilities in the home D. Assess the client's ability to participate in self-care and/or the reliability of a caregiver

D The cognitive ability of the client as well as the availability and reliability of a caregiver must be assessed to determine if home care is a feasible option. The other approaches are correct and would be pursued after this initial approach.

A client receiving chemotherapy has developed sores in the mouth and asks the nurse why this has happened. How should the nurse respond? A. "This always happens with chemotherapy." B. "You need to have better oral hygiene." C. "It is a sign that the medication is working." D. "The cells in the mouth are sensitive to the chemotherapy."

D The epithelial cells in the mouth are very sensitive to chemotherapy due to their high rate of cell turnover.

A nurse is planning to administer otic drops to a 6 year-old child. Which action is part of the correct procedure? A. Place several drops in the outer ear B. Assist the child to lie on the affected side afterward C. Insert cotton in the outer ear after giving medication D. Hold the pinna up and back to instill the drops

D The external auditory canal should be straightened by gently pulling the pinna up and back for otic drop administration. In children who are under three years of age, the pinna should be pulled down and back.

A 3 year-old child is treated in the emergency department after ingestion of one ounce of a liquid narcotic. What action should the nurse perform first? A. Prepare for gastric lavage B. Start the ordered intravenous fluids C. Obtain blood and urine samples D. Assess airway, breathing, circulation and level of consciousness

D The first step in treatment of a toxic exposure or ingestion is to assess the airway, breathing and circulation (ABCs), particularly when the substance is known to cause CNS depression, such as a narcotic. The level of consciousness will also be an important indicator of the CNS-depressant effects of the medication. The other actions are correct and also important, but they are not the first priority.

As a nurse is speaking with a group of teens about cancer chemotherapy. The nurse anticipates that this group will be most interested in discussing which of the following side effects of the treatment? A. Fatigue B. Diarrhea C. Mouth sores D. Hair loss

D The major concern in adolescence is body image. Thus, hair loss would be the most disturbing side effect for this group of clients.

The nurse is caring for a client who is unconscious and receiving gastric tube feedings. Which assessment finding requires an immediate action from the nurse? A. Formula residual volume 100 mL B. Urine output of 250 mL in the past eight hours C. Decreased bowel sounds in all quadrants D. Decreased breath sounds in the right lower lobe

D The most common problem associated with enteral feedings is aspiration with resulting atelectasis and pneumonia. A nursing action should be to maintain clients at a minimum of 30 degrees of head elevation during feedings and up to two hours afterwards. The nurse should verify tube placement prior to each feeding or every four to eight hours if the client receives a continuous feeding.

A couple experience the loss of a 7 month-old fetus. In planning for discharge, what should the nurse emphasize? A. To plan for another pregnancy within two years and maintain physical health B. To seek causes for the fetal death and come to some safe conclusion C. To focus on the other healthy children and move through the loss D. To discuss feelings with each other and use grief resources

D To communicate in a therapeutic manner, the nurse's goal is to help the couple begin the grief process by suggesting they talk to each other, seek family, friends and support groups to listen to their feelings. To look for causes or set a time to plan another pregnancy is inappropriate.

Prior to discharge home, a treatment plan is being developed for a client with severe arthritis. What is the most important part of the treatment plan? A. Anticipate side effects of therapy B. Ensure compliance with medications C. Support coping with limitations D. Maintain and preserve functional status

D To maintain quality of life and to ensure safety, the treatment plan must emphasize maintaining functional status. The client's ability to perform activities of daily living (ADLs) and instrumental activities of daily living (IADLs) should be assessed; if needed, referrals for physical and/or occupational therapy can be made. All clients should understand the purpose of any prescribed medications, as well as how and when to take them, expected side effects and possible adverse effects.

A nurse is assigned to care for a comatose client. The client is diagnosed with type 1 diabetes and has an IV infusion of insulin running. Which task would be most appropriate to assign to an unlicensed assistive person (UAP)? A. Obtain a peripheral blood glucose reading B. Determine if special skin care is needed when reddened areas are identified C. Answer questions from the client's spouse about the plan of care D. Measure and document the hourly output in a urine collection bag

D UAP perform routine and predictable tasks, such as measuring and recording urine output. For a comatose client diagnosed with type 1 diabetes, it would be inappropriate for the UAP to obtain a peripheral blood glucose measurements. The RN must closely monitor the client to prevent hypoglycemia and follow orders for titrating the IV insulin based on changing insulin needs. The nurse is responsible for providing information about the plan of care to the client's family. Assessing skin care needs is a function of the nurse.

The nurse is teaching a client about precautions while taking warfarin. The nurse should instruct the client to avoid foods with excessive amounts of which nutrient? A. Calcium B. Vitamin E C. Iron D. Vitamin K

D Vitamin K is an essential vitamin required for blood clotting. Eating foods with excessive amounts of Vitamin K may alter anticoagulant effects. Foods highest in vitamin K include (dried and fresh) herbs, dark leafy greens, scallions, brussel sprouts, broccoli, chili powder, prunes, asparagus and cabbage.


Kaugnay na mga set ng pag-aaral

CH. 9 - PRODUCTION AND OPERATIONS MGMT

View Set

NU271 PrepU: Disorders of the Immune Response (week 9)

View Set

Civil Procedure - Professor MCQ's

View Set

Simulation Lab 11.1: Module 11 Harden PC with Group Policy Editor

View Set